Dino Dio

Algebra Level 5

a , b , c , d a, b, c, d are integers such that

{ a + b + c + d = 0 ( a b c d ) ( a c b d ) ( a d b c ) + 52 8 2 = 0 \begin{cases} a + b + c + d = 0 \\ (ab-cd)(ac-bd)(ad-bc) + 528 ^2 =0 \\ \end{cases}

What is the maximum possible value of a a ?


The answer is 265.

This section requires Javascript.
You are seeing this because something didn't load right. We suggest you, (a) try refreshing the page, (b) enabling javascript if it is disabled on your browser and, finally, (c) loading the non-javascript version of this page . We're sorry about the hassle.

4 solutions

Happy Melodies
Dec 3, 2013

When we see questions with a + b + c + d = 0 a+b+c+d = 0 or something alike, we always think to express one of the terms as the others. In this case, let a = ( b + c + d ) a = -(b+c+d) ----------- (1).

Substitute (1) into the second given equation and we get,

( b 2 + b c + c d + b d ) ( c 2 + b c + c d + b d ) ( d 2 + b c + c d + b d ) + 52 8 2 = 0 -(b^{2} +bc + cd + bd)(c^{2} +bc + cd + bd)(d^{2} +bc + cd + bd) + 528^{2} = 0

Then, notice that we can factorise this into: ( b + d ) ( b + c ) ( c + b ) ( c + d ) ( c + d ) ( b + d ) = 52 8 2 -(b+d)(b+c)(c+b)(c+d)(c+d)(b+d) = - 528^{2}

From this, it is clear that ( b + d ) 2 ( b + c ) 2 ( c + d ) 2 = 52 8 2 (b+d)^{2}(b+c)^{2}(c+d)^{2} = 528^{2} . Thus, ( b + d ) ( b + c ) ( c + d ) = 528 (b+d)(b+c)(c+d) = 528 or 528 -528 .

From (1), notice for a a to be the maximum, b + c + d b+c+d must be the minimum, (this is the same as b + c + d |b+c+d| being the maximum). Hence, for max a a , ( b + d ) ( b + c ) ( c + d ) = 528 (b+d)(b+c)(c+d) = -528 --------(2).

Furthermore, observe that the sum of the terms in the product of (2) equal b + d + b + c + c + d = 2 ( b + c + d ) = 2 a b+d+b+c+c+d = 2(b+c+d) = -2a . Remembering that for max. a a , the absolute value of b + c + d b+c+d needs to be maximise, hence absolute value of 2 ( b + c + d ) 2(b+c+d) needs to be the maximum, which in our question is simply 528 + 1 + 1 = 530 528 +1+1 = 530 . [Note 2 ( b + c + d ) = 530 |2(b+c+d)| = 530 when unordered values of ( b , c , d ) = ( 264 , 264 , 263 ) (b,c,d) = (-264, -264, 263) . I got these values by letting b + c = 528 b+c = - 528 and c + d = b + d = 1 c+d=b+d = -1 . But this isn't important to solve the question, since all we want to know is the maximum absolute value of (b+c+d)).

Thus, the minimum value of b + c + d = 530 2 = 265 b+c+d = \frac{-530}{2} = -265 , and maximum a = ( b + c + d ) = 265 a = -(b+c+d) = \boxed{265} .

Generalisation

When faced with the same question (with different values of the 52 8 2 = x 2 528^{2} = x^{2} ), the max. a = x + 1 + 1 2 a = \frac{x+1+1}{2}

Very well written solution!!

Jit Ganguly - 7 years, 6 months ago

Log in to reply

Thank you very much :)

Happy Melodies - 7 years, 6 months ago

Thanks for the clear explanation :)

Pinku Deb Nath - 7 years, 6 months ago

Log in to reply

Your welcome :)

Happy Melodies - 7 years, 6 months ago

so when the product of terms= sum of terms, it is a maximum?

Henry Zeng - 7 years, 6 months ago

Log in to reply

I am not sure if I have understood your question correctly, but not really. Like, in this case, the sum of terms(=-530) is not equal to the product of terms (=-528).

Happy Melodies - 7 years, 6 months ago

Log in to reply

Yeah, i didn't know how you got the sum of terms to equal 530 if 'a' is maximized. Why is this?

Henry Zeng - 7 years, 6 months ago

Log in to reply

@Henry Zeng I got the sum of the terms ( b + c ) , ( b + d ) (b+c), (b+d) and ( c + d ) (c+d) to be 530 -530 when one of these 3 terms equals 528 -528 and the other 2 terms are equal to 1 -1 , [Since 528 1 1 = 528 -528 * -1 *-1 = -528 ]

So how did I derive the above? Why is it that a a is maximised when the sum of the 3 terms is 530 -530 . This is because for a a to reach its maximum, the absolute value of b + c + d b+c+d needs to be maximise as well, since a = ( b + c + d ) a = -(b+c+d) . Note that the maximum value of 2 ( b + c + d ) = \mid 2(b+c+d) \mid = the maximum value of the absolute value of the sum of the 3 terms = 530 = 530 . This is because as a general rule, to obtain the largest sum, we need a very large number added to a small number, rather than two "middle-values" (not big nor small) numbers add together. For example, in the product 100 = 100 1 = 10 10 100 = 100*1 = 10*10 , the sum of terms 100 + 1 = 101 > 10 + 10 = 20 100 +1 = 101 > 10+10 = 20 .

Applying the above to our question, we obtain the largest possible absolute value a term can take in our product is 528 528 .

P.S. Be careful of the notations: Absolute value means always positive.

Happy Melodies - 7 years, 6 months ago

Log in to reply

@Happy Melodies Alright. Thanks. I guess the general rule bit is more about logic

Henry Zeng - 7 years, 6 months ago
Jared Low
Dec 2, 2013

Let us generalise this problem, replacing 528 528 with 2 n 2n , where n n is a positive integer. We shall show that the maximum possible value of a a is n + 1 n+1

From the first equation, we have a = ( b + c + d ) a=-(b+c+d) . Substituting this value of a a into the second equation, we have:

( b 2 b c d b c d ) ( c 2 d b c d b c ) ( d 2 c d b c d b ) + n 2 = 0 (-b^2-bc-db-cd)(-c^2-db-cd-bc)(-d^2-cd-bc-db)+n^2=0

Note that we can factorise b 2 b c d b c d = ( b 2 + b c + d b + c d ) = ( b + c ) ( b + d ) -b^2-bc-db-cd=-(b^2+bc+db+cd)=-(b+c)(b+d)

Thus we can factorise, with analagous reasoning on the latter two terms, the equation into:

( b + c ) ( b + d ) ( c + d ) ( c + b ) ( d + b ) ( d + c ) = n 2 (b+c)(b+d)(c+d)(c+b)(d+b)(d+c)=n^2

Which can be further simplified as:

( b + c ) 2 ( c + d ) 2 ( d + b ) 2 = n 2 (b+c)^2(c+d)^2(d+b)^2=n^2

We wish to find the maximum possible value of a, which occurs when ( b + c + d ) (b+c+d) is at its minimum value, or when ( b + c ) + ( c + d ) + ( d + c ) = 2 ( b + c + d ) (b+c)+(c+d)+(d+c)=2(b+c+d) is at its minimum value. This would occur with ( b + c ) , ( c + d ) , ( d + b ) (b+c),(c+d),(d+b) being all negative, which would then result in ( b + c ) + ( c + d ) + ( d + b ) = n (b+c)+(c+d)+(d+b)=-n Assume to the contrary we have some ( b + c ) , ( c + d ) , ( d + b ) (b+c),(c+d),(d+b) where at least one of them has a positive value, then ( b + c + d ) (b+c+d) will have a lower value simply by making said value negative.

We now show that the minimum value of b + c + d b+c+d is obtained when [ ( b + c ) , ( c + d ) , ( d + b ) ] = [ 2 n , 1 , 1 ] [(b+c),(c+d),(d+b)]=[-2n, -1, -1] or any other permutation thereof, giving b + c + d = n b+c+d=-n , which works with one possible solution being ( a , b , c , d ) = ( n + 1 , n , n , n 1 ) (a,b,c,d)=(n+1,-n,-n,n-1) by establishing the following claim:

Claim : Across all triplets of integers x , y , z x,y,z where their product is m m , the largest possible value for x + y + z x+y+z is m + 2 m+2 , which occurs for ( x , y , z ) = ( m , 1 , 1 ) (x,y,z)=(m,1,1) or permutatuins thereof.

Proof : We know that for all pairs of integers ( x , y ) (x,y) with their product m m , the largest possible product occurs when ( x , y ) = ( m , 1 ) (x,y)=(m,1) . By fixing one of the integers in our triplet ( x , y , z ) (x,y,z) , we can see that at least one of the other two integers must have the value 1 1 to maximise their sum. With said integer with value of 1 1 , the other two must follow to have values n , 1 n, 1 , which completes the proof.

For our case involving ( b + c ) ( c + d ) ( d + b ) = n (b+c)(c+d)(d+b)=-n and us wanting the minimum value for ( b + c ) + ( c + d ) + ( d + c ) = 2 ( b + c + d ) (b+c)+(c+d)+(d+c)=2(b+c+d) , we simply invert the signs for ( b + c ) , ( c + d ) , ( d + b ) (b+c),(c+d),(d+b) , giving us b + c + d = 2 n 2 2 = n 1 b+c+d=\frac{-2n-2}{2}=-n-1 , with a = ( b + c + d ) = n + 1 a=-(b+c+d)=n+1

For this specific example of our generalised problem with n = 528 2 = 264 n=\frac{528}{2}=264 , we thus have our answer to be 264 + 1 = 265 264+1=\fbox{265}

Note that at the beginning of this solution it is specified that we had numbers of form 2 n 2n , i.e. even numbers, to replace the value of 528 528 . We are unable to find any solution in integers for ( a , b , c , d ) (a,b,c,d) if said number was odd, as the values ( b + c ) , ( c + d ) , ( d + b ) (b+c),(c+d),(d+b) would all be odd and consequently 2 ( b + c + d ) 2(b+c+d) would be odd as well, a contradiction.

BTW you wrote: ( b + c ) + ( c + d ) + ( d + c ) = 2 ( b + c + d ) (b+c)+(c+d)+(d+c)=2(b+c+d) instead of: ( b + c ) + ( c + d ) + ( d + b ) = 2 ( b + c + d ) (b+c)+(c+d)+(d+b)=2(b+c+d)

Patrick Chen - 7 years, 6 months ago

Log in to reply

Thanks for the heads up.

Jared Low - 7 years, 6 months ago
David Altizio
Dec 6, 2013

What an interesting problem. I was expecting this to be a lot uglier than it ended up being, although this official solution is nowhere near how messy my scratch paper became.

We first attempt to make that nasty product simpler. Note that using the condition a + b + c + d = 0 a+b+c+d=0 we can see that

a b c d = a b + a c a c c d = a ( b + c ) c ( a + d ) = a ( b + c ) + c ( b + c ) = ( a + c ) ( b + c ) . \begin{aligned}ab-cd&=ab+ac-ac-cd=a(b+c)-c(a+d)\\&=a(b+c)+c(b+c)=(a+c)(b+c).\end{aligned}

Similarly, a c b d = ( a + d ) ( b + d ) ac-bd=(a+d)(b+d) and a d b c = ( a + b ) ( d + b ) ad-bc=(a+b)(d+b) . Therefore, the second equation becomes

( a + b ) ( a + c ) ( a + d ) ( b + c ) ( b + d ) ( c + d ) = 52 8 2 . (a+b)(a+c)(a+d)(b+c)(b+d)(c+d)=-528^2.
Noting once again that a + b = ( c + d ) a+b=-(c+d) , a + c = ( b + d ) a+c=-(b+d) , and a + d = ( b + c ) a+d=-(b+c) , we can simplify this product to

( a + b ) 2 ( a + c ) 2 ( a + d ) 2 = 52 8 2 ( a + b ) ( a + c ) ( a + d ) = 528. \begin{aligned}-(a+b)^2(a+c)^2(a+d)^2&=-528^2\\\implies (a+b)(a+c)(a+d)&=528.\end{aligned}

It remains to maximize the sum of the three factors on the left hand side, as ( a + b ) + ( a + c ) + ( a + d ) = 3 a + b + c + d = 2 a (a+b)+(a+c)+(a+d)=3a+b+c+d=2a , and maximizing this sum will in turn maximize a a .

LEMMA: Suppose x x , y y , and z z are positive integers such that x y z = c xyz=c . Then x + y + z c + 2 x+y+z\leq c+2 .

Proof. Note that since x 1 x\geq 1 and y 1 y\geq 1 , we have

( x 1 ) ( y 1 ) 0 x + y x y + 1. (x-1)(y-1)\geq 0\implies x+y\leq xy+1.

Therefore, by applying this repeatedly we have

x + y + z x y + z + 1 x y z + 2 = c + 2 , x+y+z\leq xy+z+1\leq xyz+2=c+2,

as desired. Equality holds when ( x , y , z ) = ( c , 1 , 1 ) (x,y,z)=(c,1,1) or permutations. \blacksquare

Applying this lemma to our original problem problem, we have

2 a 528 + 1 + 1 = 530 a 265 . 2a\leq 528+1+1=530\implies a\leq \boxed{265}.

Ronald Overwater
Jan 10, 2014

Using a = ( b + c + d ) a=-(b+c+d) the second relation can be rewritten to

( b + c ) 2 ( b + d ) 2 ( c + d ) 2 + 52 8 2 = 0 -(b+c)^2(b+d)^2(c+d)^2 + 528^2 = 0

So we have ( b + c ) ( b + d ) ( c + d ) = 528 |(b+c)(b+d)(c+d)|=528 .

Using x = b + c x=b+c , y = b + d y=b+d and z = c + d z=c+d we get:

x y z = 528 |xyz|=528 and 2 a = ( x + y + z ) 2a=-(x+y+z) .

Maximum for 2 a = 530 2a=530 when e.g. ( x , y , z ) = ( 528 , 1 , 1 ) (x,y,z)=(-528,-1,-1) ,

so a m a x = 530 2 = 265 a_{max} = \frac{530}{2} = \fbox{265}

This question seems to be easy but is not so simple to solve! Nice solution.

Answer: 265 \boxed{265}

Saurabh Mallik - 7 years, 2 months ago

0 pending reports

×

Problem Loading...

Note Loading...

Set Loading...